Partially answer Apostol exercises I 3.12.

Include basic ideas around rational numbers.
finite-set-exercises
Joshua Potter 2023-04-21 07:17:21 -06:00
parent 563217c915
commit 3bc07ffeb5
4 changed files with 141 additions and 22 deletions

View File

@ -138,8 +138,7 @@ If there exists some `k` satisfying the description in the above `def`, then `Q`
is measurable.
-/
axiom exhaustion_exists_unique_imp_measurable (Q : Set ℝ²)
: (∃ k : , forall_subset_between_step_imp_le_between_area k Q ∧
(∀ x : , forall_subset_between_step_imp_le_between_area x Q → x = k))
: (∃! k : , forall_subset_between_step_imp_le_between_area k Q)
→ Q ∈ 𝓜
/--
@ -152,6 +151,6 @@ axiom exhaustion_exists_unique_imp_area_eq (Q : Set ℝ²)
: ∃ k : ,
(h : forall_subset_between_step_imp_le_between_area k Q ∧
(∀ x : , forall_subset_between_step_imp_le_between_area x Q → x = k))
→ area (exhaustion_exists_unique_imp_measurable Q ⟨k, h⟩) = k
→ area (exhaustion_exists_unique_imp_measurable Q ⟨k, h⟩) = k
end Real

View File

@ -3,6 +3,14 @@ Exercises I 3.12
A Set of Axioms for the Real-Number System
-/
import Mathlib.Algebra.Order.Floor
import Mathlib.Data.PNat.Basic
import Mathlib.Data.Real.Basic
import Mathlib.Data.Real.Sqrt
import Mathlib.Tactic.LibrarySearch
import Apostol.Chapter_I_3
import Bookshelf.Real.Rational
-- ========================================
-- Exercise 1
@ -11,7 +19,17 @@ A Set of Axioms for the Real-Number System
-- at least one real `z` satisfying `x < z < y`.
-- ========================================
-- # TODO
example (x y : ) (h : x < y) : ∃ z, x < z ∧ z < y := by
have ⟨z, hz⟩ := exists_pos_add_of_lt' h
refine ⟨x + z / 2, ⟨?_, ?_⟩⟩
· have hz' : z / 2 > 0 := by
have hr := div_lt_div_of_lt (show (0 : ) < 2 by simp) hz.left
rwa [zero_div] at hr
exact (lt_add_iff_pos_right x).mpr hz'
· have hz' : z / 2 < z := div_lt_self hz.left (show 1 < 2 by norm_num)
calc x + z / 2
_ < x + z := (add_lt_add_iff_left x).mpr hz'
_ = y := hz.right
-- ========================================
-- Exercise 2
@ -20,7 +38,8 @@ A Set of Axioms for the Real-Number System
-- such that `m < x < n`.
-- ========================================
-- # TODO
example (x : ) : ∃ m n : , m < x ∧ x < n := by
refine ⟨x - 1, ⟨x + 1, ⟨?_, ?_⟩⟩⟩ <;> norm_num
-- ========================================
-- Exercise 3
@ -28,7 +47,12 @@ A Set of Axioms for the Real-Number System
-- If `x > 0`, prove that there is a positive integer `n` such that `1 / n < x`.
-- ========================================
-- # TODO
example (x : ) (h : x > 0) : ∃ n : +, 1 / n < x := by
have ⟨n, hn⟩ := @Real.exists_pnat_mul_self_geq_of_pos x 1 h
refine ⟨n, ?_⟩
have hr := mul_lt_mul_of_pos_right hn (show 0 < 1 / ↑↑n by norm_num)
conv at hr => arg 2; rw [mul_comm, ← mul_assoc]; simp
rwa [one_mul] at hr
-- ========================================
-- Exercise 4
@ -39,7 +63,13 @@ A Set of Axioms for the Real-Number System
-- `⌊5 / 2⌋ = 2`, `⌊-8/3⌋ = -3`.
-- ========================================
-- # TODO
example (x : ) : ∃! n : , n ≤ x ∧ x < n + 1 := by
let n := Int.floor x
refine ⟨n, ⟨?_, ?_⟩⟩
· exact ⟨Int.floor_le x, Int.lt_floor_add_one x⟩
· intro y hy
rw [← Int.floor_eq_iff] at hy
exact Eq.symm hy
-- ========================================
-- Exercise 5
@ -48,7 +78,19 @@ A Set of Axioms for the Real-Number System
-- `n` which satisfies `x ≤ n < x + 1`.
-- ========================================
-- # TODO
example (x : ) : ∃! n : , x ≤ n ∧ n < x + 1 := by
let n := Int.ceil x
refine ⟨n, ⟨?_, ?_⟩⟩
· exact ⟨Int.le_ceil x, Int.ceil_lt_add_one x⟩
· simp only
intro y hy
suffices y - 1 < x ∧ x ≤ y by
rw [← Int.ceil_eq_iff] at this
exact Eq.symm this
apply And.intro
· have := (sub_lt_sub_iff_right 1).mpr hy.right
rwa [add_sub_cancel] at this
· exact hy.left
-- ========================================
-- Exercise 6
@ -59,7 +101,8 @@ A Set of Axioms for the Real-Number System
-- are *dense* in the real-number system.
-- ========================================
-- # TODO
example (x y : ) (h : x < y) : ∃ r : , x < r ∧ r < y := by
sorry
-- ========================================
-- Exercise 7
@ -68,7 +111,25 @@ A Set of Axioms for the Real-Number System
-- `xy`, `x / y`, and `y / x` are all irrational.
-- ========================================
-- # TODO
example (x : ) (hx : x ≠ 0) (y : ) (hy : irrational y)
: irrational (x + y) := by
sorry
example (x : ) (hx : x ≠ 0) (y : ) (hy : irrational y)
: irrational (x - y) :=
sorry
example (x : ) (hx : x ≠ 0) (y : ) (hy : irrational y)
: irrational (x * y) :=
sorry
example (x : ) (hx : x ≠ 0) (y : ) (hy : irrational y)
: y ≠ 0 → irrational (x / y) :=
sorry
example (x : ) (hx : x ≠ 0) (y : ) (hy : irrational y)
: irrational (y / x) :=
sorry
-- ========================================
-- Exercise 8
@ -76,7 +137,10 @@ A Set of Axioms for the Real-Number System
-- Is the sum or product of two irrational numbers always irrational?
-- ========================================
-- # TODO
-- No. Here is a counterexample.
example (hx : x = Real.sqrt 2): irrational x ∧ rational (x * x) := by
sorry
-- ========================================
-- Exercise 9
@ -86,7 +150,8 @@ A Set of Axioms for the Real-Number System
-- infinitely many.
-- ========================================
-- # TODO
example (x y : ) (h : x < y) : ∃ z : , irrational z ∧ x < z ∧ z < y := by
sorry
-- ========================================
-- Exercise 10
@ -94,17 +159,46 @@ A Set of Axioms for the Real-Number System
-- An integer `n` is called *even* if `n = 2m` for some integer `m`, and *odd*
-- if `n + 1` is even. Prove the following statements:
--
-- (a) An integer cannot be both even and odd.
-- (b) Every integer is either even or odd.
-- (c) The sum or product of two even integers is even. What can you say about
-- the sum or product of two odd integers?
-- (d) If `n²` is even, so is `n`. If `a² = 2b²`, where `a` and `b` are
-- integers, then both `a` and `b` are even.
-- (e) Every rational number can be expressed in the form `a / b`, where `a` and
-- `b` are integers, at least one of which is odd.
-- ========================================
-- # TODO
def is_even (n : ) := ∃ m : , n = 2 * m
def is_odd (n : ) := is_even (n + 1)
-- ----------------------------------------
-- (a) An integer cannot be both even and odd.
-- ----------------------------------------
example (n : ) : is_even n = ¬ is_odd n := sorry
-- ----------------------------------------
-- (b) Every integer is either even or odd.
-- ----------------------------------------
example (n : ) : is_even n is_odd n := sorry
-- ----------------------------------------
-- (c) The sum or product of two even integers is even. What can you say about
-- the sum or product of two odd integers?
-- ----------------------------------------
example (m n : ) : is_even m ∧ is_even n → is_even (m * n) := sorry
example (m n : ) :
(∃ m n : , is_odd m ∧ is_odd n ∧ is_even (m * n)) ∧
(∃ m n : , is_odd m ∧ is_odd n ∧ is_odd (m * n)) :=
sorry
-- ----------------------------------------
-- (d) If `n²` is even, so is `n`. If `a² = 2b²`, where `a` and `b` are
-- integers, then both `a` and `b` are even.
-- ----------------------------------------
example (n : ) (h : is_even (n ^ 2)) : is_even n := sorry
example (a b : ) (h : a ^ 2 = 2 * b ^ 2) : is_even a ∧ is_even b := sorry
-- ========================================
-- Exercise 11
@ -116,16 +210,27 @@ A Set of Axioms for the Real-Number System
-- contradiction.]
-- ========================================
-- # TODO
example : ¬ ∃ n : , rational n → n ^ 2 = 2 := sorry
-- ========================================
-- Exercise 12
--
-- The Archimedean property of the real-number system was deduced as a
-- consequence of the least-upper-bound axiom. Prove that the set of rational
-- numbers satisfies the Archimedean property but not he least-upper-bound
-- numbers satisfies the Archimedean property but not the least-upper-bound
-- property. This shows that the Archimedean property does not imply the
-- least-upper-bound axiom.
-- ========================================
-- # TODO
/--
Shows the set of rational numbers satisfies the Archimedean property.
-/
theorem exists_pnat_mul_self_geq_of_pos {x y : }
: x > 0 → ∃ n : +, n * x > y := sorry
/--
Show the Archimedean property does not imply the least-upper-bound axiom.
-/
example (S : Set ) (hne : S.Nonempty) (hbdd : BddAbove S)
: ¬ ∃ x, IsLUB S x :=
sorry

View File

@ -1,5 +1,6 @@
import Bookshelf.Real.Basic
import Bookshelf.Real.Function
import Bookshelf.Real.Geometry
import Bookshelf.Real.Rational
import Bookshelf.Real.Sequence
import Bookshelf.Real.Set

View File

@ -0,0 +1,14 @@
import Bookshelf.Real.Basic
/--
Assert that a real number is rational.
Note this does *not* require the found rational to be in reduced form. Members
of `` expect this (by proving the numerator and denominator are coprime).
-/
def rational (x : ) := ∃ a : , ∃ b : , b ≠ 0 ∧ x = a / b
/--
Assert that a real number is irrational.
-/
def irrational (x : ) := ¬ rational x